Inscription / Connexion Nouveau Sujet
Niveau Licence Maths 1e ann
Partager :

idéal

Posté par
mickey12
14-06-11 à 15:05

-Montrer que l'idéal de l'anneau de polynome Z[X] engendré par 2 et X n'est pas principal.
Pour cela, je prend les éléments 2+50X et 2+51X (deux éléments du gpe engendré par 2 et X) et je veux montrer "qu'ils n'appartiennent pas à un mm idéal principal".Je suppose que si donc il existe k et k' tels que:
2+50X=k(a+bX) (1)
2+51X=k'(a+bX) (2)
j'ai tenté de soustraire (1) à (2)ais je n'aboutit à rien... je pense qu'il n'est pas bon de mettre les inconnus a et b....
Je n'arrive pas à me sortir de ce problème...
Autre question:
-Montrer(en utilisant la division euclidienne) que si K est un corps, alors tous les idéaux de l'anneau de polynome K[X] sont principaux. Comment démarrer?Besoin d'un coup de pouce...

Posté par
Camélia Correcteur
re : idéal 14-06-11 à 15:25

Bonjour (encore moi)

Ta méthode n'est pas très appropriée... autant faire ce que l'on te demande. Si l'idéal \cal I engendré par 2 et X était principal engendré par un élément D\in{\cal I}, alors 2 et X qui sont dedans seraient multiples de D ce qui entraine D=\pm 1. Mais on ne peut en aucun cas écrire 1=2P(X)+XQ(X) parce que on aurait 2P(0)=1. Donc contradiction!

2) Prends un idéal I non réduit à 0. Soit A(X) un polynôme de degré m dans I, avec m minimal. Prends P dans I et fais la division euclidienne de P par A. Tu devrais pouvoir montrer que P est multiple de A.

Posté par
mickey12
re : idéal 15-06-11 à 10:22

merci pour le coup de pouce!
Dans le 1),si j'ai bien compris, il y a contradiction car P n'appartient pas à Z[X]?

Posté par
Camélia Correcteur
re : idéal 15-06-11 à 14:12

Oui, bien sur aucun polynôme à coefficients entiers ne peut vérifier P(0)=1/2.

Posté par
mickey12
re : idéal 16-06-11 à 10:20

par contre pour le 2), je vois pas trop. "m minimal" ?????
En fait tu veux montrer que I est engendré par A(X)?

Posté par
Camélia Correcteur
re : idéal 16-06-11 à 14:07

m le plus petit possible! Oui, il faut montrer que I est engendré par un polynôme de degré minimal.

Posté par
mickey12
re : idéal 16-06-11 à 14:29

non mais ca me gêne vraiment ce terme de minimal...
si on fait la division de P par A, on aura:
P(X)=Q(X)A(X)+R(X) avec deg(R)<deg(A). On doit alors montrer que "R=0"... pour avoir P multiple de A mais on sait juste que P est dans I et que A aussi avec A de degré minimal...

Posté par
Camélia Correcteur
re : idéal 16-06-11 à 14:32

Eh bien, voilà, tu y es.

Donc A est de degré minimal m. Tu écris P=QA+R avec deg(R) < m ou R=0. Mais R=P-QA, donc R est dans I. Comme on ne peut pas avoir deg(R) < m, on a forcément R=0.

Posté par
mickey12
re : idéal 17-06-11 à 18:19

ok merci bcp camélia!

Posté par
mickey12
re : idéal 18-06-11 à 14:52

Par contre, ou utilise t-on le fait que K est un corps????

Posté par
Camélia Correcteur
re : idéal 18-06-11 à 15:01

La division euclidienne n'est définie que dans K[X] avec K corps.



Vous devez être membre accéder à ce service...

Pas encore inscrit ?

1 compte par personne, multi-compte interdit !

Ou identifiez-vous :


Rester sur la page

Inscription gratuite

Fiches en rapport

parmi 1675 fiches de maths

Désolé, votre version d'Internet Explorer est plus que périmée ! Merci de le mettre à jour ou de télécharger Firefox ou Google Chrome pour utiliser le site. Votre ordinateur vous remerciera !